Show operator is compact/symmetric

  • Thread starter Thread starter member 428835
  • Start date Start date
  • Tags Tags
    Operator
Click For Summary
SUMMARY

The discussion centers on the properties of the compact symmetric operator ##K## defined on the space ##L^{2}[0,1]##, specifically regarding its eigenvalues and eigenfunctions. The participants establish that ##K## is compact due to the uniform convergence of a subsequence of continuous functions derived from a bounded sequence in ##L^{2}[0,1]##. They explore the implications of the operator's kernel and the conditions under which eigenfunctions exist, concluding that while there may be finitely many non-zero eigenvalues, infinitely many eigenfunctions can correspond to the zero eigenvalue.

PREREQUISITES
  • Understanding of compact operators in functional analysis
  • Familiarity with the properties of symmetric operators
  • Knowledge of eigenvalues and eigenfunctions in the context of integral operators
  • Proficiency in the concepts of uniform convergence and equicontinuity
NEXT STEPS
  • Study the properties of compact operators in Hilbert spaces
  • Learn about the spectral theorem for compact symmetric operators
  • Investigate the relationship between the kernel of an operator and its eigenvalues
  • Explore examples of compact symmetric operators and their eigenvalue distributions
USEFUL FOR

Mathematicians, particularly those specializing in functional analysis, graduate students studying operator theory, and researchers exploring the properties of compact symmetric operators in Hilbert spaces.

member 428835
Homework Statement
Consider the operator ##K:L_2[0,1]\to L_2[0,1]## given by ##Kf(x) = \int_0^1(x+y)f(y)\, dy##
1) Prove that ##K## is a compact symmetric operator.
2) Find all eigenvalues of ##K##.
3) Manually verify that eigenvectors of ##K## corresponding to different eigenvalues are pairwise orthogonal.
Relevant Equations
Nothing comes to mind
1) To show that ##K## is compact let ##\{ f_{n} \}_{n=1}^{\infty}## be a bounded sequence in ##L^{2}[0,1]## with ##\|f_{n}\| \le M##. For every ##\epsilon > 0##, there exists ##\delta > 0## such that ##|k(x,y)-k(x',y')| < \epsilon## whenever ##|x-x'|+|y-y'| < \delta##. Therefore, ##\{ Kf_{n}\}## is a sequence of continuous functions for which
$$
|Kf_{n}(x)-Kf_{n}(x')| \le \int_{0}^{1}|k(x,y)-k(x',y)||f_{n}(y)|\,dy \\
\le \epsilon \int_{0}^{1}|f_{n}(y)|\,dy \le \epsilon\|1\|\|f_{n}\|
\le M\epsilon,\;\;\; |x-x'| < \delta
$$
where ##k(x,y) = x+y##. Thus ##\{ Kf_{n} \}## is an equicontinuous family of continuous functions on ##[0,1]##. So, there exists a subsequence ##\{ Kf_{n_{k}}\}## that converges uniformly to a continuous function ##g##. Since uniform convergence implies convergence in ##L^{2}[0,1]##, it follows that ##\{ Kf_{n_{k}}\}## converges in ##L^{2}[0,1]##. Therefore ##K## is compact because the image of a bounded sequence always contains a convergent subsequence.

2) no clue. I'm thinking ##\det (\lambda I - K) = 0##? Then ##\int_0^1(x+y) f(y)\, dy = \lambda f(x)##? It seems obvious to me that ##f## must be a linear function, so that ##f(x) = a x + b##. Then I think $$\int_0^1(x+y)(ay+b)\,dy = \lambda(ax+b)\implies\\
a/3 + b/2 + (a/2 + b) x = \lambda b + \lambda a x.$$ Weighting equations implies ##\lambda_1 = 1/6 (3 - 2 \sqrt 3)## when ##b = -(a/\sqrt 3 )## or ##\lambda_2 = 1/6 (3 + 2 \sqrt 3)## when ##b = (a/\sqrt 3 )##. Then our eigenvalues are determined and eigenfunctions are determined up to the constant ##a##. Is the right so far? If so, what's next?

3) I need 2) to even attempt.
 
Last edited by a moderator:
Physics news on Phys.org
I suggest simplifying your formula for (Kf)(x). It becomes obvious that (Kf)(x) is always linear, with linear coefficient the integral of f and constant term the integral of y. f(y). Hence you are right that any eigenfunction f must also be linear. Then just compute as you have done to find all eigenvalues (I have not checked fully your computations). (The determinant seems useless in infinite dimensions.)

I do not know what you mean by "weighting equations", but setting coefficients equal on both sides seems to give that f = ax+b is an eigenfunction iff 3b^2 = a^2, and the corresponding eigenvalue is (a+2b)/2a = (2a+3b)/6b. Is that what you got? oh yes, indeed then b = a/sqrt(3), so probably I agree.

You seem to have solved 2).

wait, what am i doing wrong? shouldn't a compact symmetric operator have a lot of eigenfunctions? maybe i am being careless about assuming ab ≠ 0. its dinnertime.

or maybe i need to determine the kernel of the operator. that looks more interesting. i seem to have been assuming the eigenvalue was ≠0.
 
Last edited:
mathwonk said:
wait, what am i doing wrong? shouldn't a compoact symmetric operator hve a lot of eigenfunctions? maybe i am being careless about assuming ab ≠ 0. its dinnertime.
I think only two is fine. I just googled and found an example here on page 3:

http://www.maths.qmul.ac.uk/~ig/MAS214/int-ops-lect.pdf
I didn't really read their technique but it looks like they recover only two eigenvalues. What do you think?
 
finitely many eigenvalues is cool, but doesn't a compact symmetric operator on L2 have an infinite number of eigenfunctions?

Oh yes, the kernel seems to be the key. Note that for any smooth function F with F(0) = F(1) = integral of F over [0,1], the derivative F' seems to be in the kernel. These are very easy to construct with F(0) = 0. e.g. F(x) = sin(2nπx). but i have not checked this.

Indeed the general theory seems to say that there are only finitely many eigenfunctions sharing the same ≠0 eigenvalue, but there can be infinitely many with eigenvalue zero. But this is not my game. It seems kind of fun though, and I used to like it back in the day.
 
If \int_0^1 f(y)\,dy = 0 then K[f] is a constant. Since <br /> K\left[ 12x - 6 \right] = x \int_0^1 \left(12y - 6\right)\,dy + \int_0^1 12y^2 - 6y\,dy<br /> = x \left( 6 - 6 \right) + \left(4 - 3\right) = 1<br /> we can construct for each such f a function <br /> g : [0,1] \to \mathbb{R} : x \mapsto f(x) - (12x - 6)K[f]<br /> which is then in the kernel of K.

I leave you to consider the case \int_0^1 yf(y)\,dy = 0, and whether every kernel function must satisfy at least one of those constraints.
 
pasmith said:
If \int_0^1 f(y)\,dy = 0 then K[f] is a constant. Since <br /> K\left[ 12x - 6 \right] = x \int_0^1 \left(12y - 6\right)\,dy + \int_0^1 12y^2 - 6y\,dy<br /> = x \left( 6 - 6 \right) + \left(4 - 3\right) = 1<br /> we can construct for each such f a function <br /> g : [0,1] \to \mathbb{R} : x \mapsto f(x) - (12x - 6)K[f]<br /> which is then in the kernel of K.

I leave you to consider the case \int_0^1 yf(y)\,dy = 0, and whether every kernel function must satisfy at least one of those constraints.
Can you expound a little please? I'm confused what relevance ##\int_0^1 yf(y)\,dy = 0## has for determining eigenvalues.
 
Question: A clock's minute hand has length 4 and its hour hand has length 3. What is the distance between the tips at the moment when it is increasing most rapidly?(Putnam Exam Question) Answer: Making assumption that both the hands moves at constant angular velocities, the answer is ## \sqrt{7} .## But don't you think this assumption is somewhat doubtful and wrong?

Similar threads

  • · Replies 3 ·
Replies
3
Views
960
  • · Replies 10 ·
Replies
10
Views
2K
Replies
20
Views
2K
Replies
6
Views
2K
  • · Replies 7 ·
Replies
7
Views
2K
Replies
4
Views
2K
Replies
6
Views
2K
  • · Replies 5 ·
Replies
5
Views
2K
Replies
3
Views
2K
  • · Replies 6 ·
Replies
6
Views
2K